Gibt es eine intuitive Erklärung dafür, warum die Lorentzkraft senkrecht zur Geschwindigkeit eines Teilchens und zum Magnetfeld ist?

Die Lorentzkraft auf ein geladenes Teilchen ist senkrecht zur Geschwindigkeit des Teilchens und dem Magnetfeld, durch das es sich bewegt. Dies geht aus der Gleichung hervor:

F = q v × B

Gibt es eine intuitive Erklärung für dieses Verhalten? Jede Erklärung, die ich gesehen habe, zeigt einfach auf die Gleichung und belässt es dabei.

Ich kann mathematisch akzeptieren, warum F wird senkrecht zu sein v und B (vorausgesetzt, die Gleichung stimmt, was sie natürlich auch ist). Aber das hilft mir nicht, mir vorzustellen, was im Grunde vor sich geht.

Der Versuch, eine Analogie zu gewöhnlichen Erfahrungen herzustellen, erscheint nutzlos; Wenn ich nach Norden durch ein nach Westen fließendes "Feld" laufen würde, würde ich nicht erwarten, plötzlich in den Himmel zu fliegen.

Ich hoffe, es gibt eine Möglichkeit, den Grund für dieses Verhalten zu visualisieren, ohne ein tiefes Verständnis der fortgeschrittenen Theorie zu haben. Leider lässt meine Suche nach einer Erklärung es so erscheinen, als müsste man es bis zu einigen weiteren Jahren des Studiums einfach als bizarr akzeptieren.

Wie üblich gibt es Leute, die übertriebene, nicht intuitive Erklärungen geben, die darauf abzielen, ihr Wissen zu demonstrieren, anstatt die Frage zu beantworten.
Es gibt diese Analogie arxiv.org/pdf/hep-th/9602081.pdf
Ich bin mir nicht sicher, nach welcher Art von Erklärung Sie suchen könnten. Dies ist ein grundlegendes Postulat, also von welchen grundlegenderen Prämissen würden Sie ausgehen wollen, um die Gleichung zu rechtfertigen?
Die Antwort von @Art Brown zitiert ein Beispiel aus Schwartz ' Prinzipien der Elektrodynamik , in dem die Notwendigkeit einer Kraft senkrecht zur Ebene der Geschwindigkeit und des Magnetfelds erklärt wird, um die elektrostatische Coulomb-Kraft auszugleichen. Aber diese nicht so einfache Erklärung ist alles andere als intuitiv, da " ...(es) absolut aus der Elektrostatik und der Lorentz-Transformation ableitbar ist. " Meiner Meinung nach ist die Suche nach einer intuitiven Erklärung vergeblich.
Übrigens, für ein ähnliches Beispiel wie in Schwartz ' Prinzipien der Elektrodynamik siehe meine Antwort hier Warum spürt eine stationäre Ladung keine Kraft von einem stromführenden Draht? .
Siehe auch die eng verwandte Frage: physical.stackexchange.com/questions/242499/…

Antworten (11)

Der Versuch, eine Analogie zu gewöhnlichen Erfahrungen herzustellen, erscheint nutzlos; Wenn ich nach Norden durch ein nach Westen fließendes "Feld" laufen würde, würde ich nicht erwarten, plötzlich in den Himmel zu fliegen.

Dies ist eine vernünftige Erwartung, da die elektrischen und Gravitationsfelder Kräfte erzeugen, die in Richtung des Feldes wirken. Versuchen wir also zu sehen, was schief geht, wenn wir ein Kraftgesetz für Magnetismus aufschreiben, das sich genauso verhält. Das erste, was wir versuchen könnten, wäre

F = q B ( 1 )

Nun, das geht nicht, denn eine solche Kraft würde sich genauso verhalten wie die elektrische Kraft, und es wäre somit die elektrische Kraft, kein separates Phänomen. Magnetische Kräfte sollen Wechselwirkungen von sich bewegenden Ladungen mit sich bewegenden Ladungen sein, also müssen wir sie eindeutig einbeziehen v auf der rechten Seite. Eine Möglichkeit, dies zu tun, wäre das Standard-Lorentz-Kraftgesetz, aber wir suchen nach einer Alternative, die in Richtung des Feldes geht. So könnten wir das aufschreiben:

F = q B | v | ( 2 )

Nehmen wir zum Beispiel an, wir haben identische Ladungen q mit einer Feder verbunden. Wenn sie im Gleichgewicht in Ruhe sitzen, sagt Gleichung (2), dass keine magnetische Kraft auf sie wirkt. Aber nehmen wir an, wir bringen sie dazu, nur ein wenig zu vibrieren. Jetzt fangen sie an, in Richtung des Magnetfelds abzuschießen. Dies verletzt die Energie- und Impulserhaltung.

Im Grunde läuft dies auf ein algebraisches Problem hinaus. Das Vektorkreuzprodukt hat die Verteilungseigenschaft ( v 1 + v 2 ) × B = v 1 × B + v 2 × B , und im Beispiel der Ladungen an einer Feder, mit der eigentlichen Lorentzkraft, garantiert dies, dass sich die magnetischen Kräfte an den beiden Ladungen aufheben. Wir brauchen diese Verteilungseigenschaft wirklich, und tatsächlich kann bewiesen werden, dass das Vektorkreuzprodukt die einzig mögliche Form der Vektormultiplikation (bis auf eine multiplikative Konstante) ist, die ein Vektorergebnis erzeugt, rotationsinvariant ist, distributiv ist und mit pendelt Skalarmultiplikation. (Siehe mein Buch http://www.lightandmatter.com/area1sn.html , Anhang 2.)

Pseudovektor-Argument

Es gibt ein intuitives Argument, aber das erste, was zu tun ist, ist das Poincare-Dual von B zu nehmen. In 3 Dimensionen gibt es einen Epsilon-Tensor ϵ ich j k was unveränderlich ist --- es ändert sich nicht unter Drehungen. Es hat ϵ 123 = 1 und alle Vertauschungen geben ein Minuszeichen, sodass der Wert von ϵ Null von zwei der Indizes ist gleich, und das Vorzeichen der Permutation, um 123 zu erhalten, wenn sie alle unterschiedlich sind. Der Epsilon-Tensor kontrahiert mit drei Vektoren v 1 , v 2 , v 3 ergibt die vorzeichenbehaftete Fläche, die von dem von ihnen gebildeten Parallelepiped aufgespannt wird. Da die vorzeichenbehaftete Fläche die Determinante der zu 3 Spalten zusammengesetzten v-Matrix ist, ändert sie ihr Vorzeichen unter Spiegelung aller drei Koordinatenachsen.

Die grundlegende Größe im Elektromagnetismus ist B μ v = ϵ μ v σ B σ , der Epsilon-Tensor kontrahiert mit B. Dieses Ding ist ein antisymmetrischer Tensor des Ranges 2. Weil die ϵ Tensor ist invariant, ein antisymmetrischer Tensor ist äquivalent zu einem Vektor unter Rotationen, aber nicht äquivalent unter Reflexionen. Der Grund dafür ist, dass, obwohl ein Vektor unter Reflexionen das Vorzeichen ändert, ein Tensor dies nicht tut. Dies gilt auch für B --- es ist ein Pseudovektor, wenn Sie einen Raum mit einem stromführenden Draht reflektieren, kehrt sich die Richtung von B nicht um.

Die Tatsache, dass B grundsätzlich ein Tensor und kein Vektor ist, bedeutet, dass es bei Wechselwirkung mit einem Teilchen mit der Geschwindigkeit v nur dann eine Kraft bilden kann, wenn einer der Indizes mit etwas kontrahiert wird. Das einzige, womit man sich zusammenziehen kann, ist die Geschwindigkeit, also bekommt man B μ v v μ als die Kraft, und das ist v × B

In der Relativitätstheorie ist dies die einzig natürliche Sache, da die E- und B-Felder zusammen einen antisymmetrischen 2-Tensor bilden und die Vier-Lorentz-Kraft dieser Tensor ist, der mit der 4-Geschwindigkeit kontrahiert ist. Diese Form ist so natürlich und intuitiv, dass sie keiner ausführlichen Begründung bedarf.

Mehr physische Wiederholung des Arguments

Das Obige klingt irgendwie formal, sagt aber nur Folgendes: Das Magnetfeld ändert das Vorzeichen nicht, wenn die Koordinaten des Raums umgekehrt werden. Um dies physikalisch zu sehen, betrachten Sie ein Solenoid mit Strom, das sich entlang der z-Achse von -a nach a erstreckt, wobei der Strom hauptsächlich in der xy-Ebene entlang jeder Wicklung verläuft, und spiegeln Sie dieses Solenoid in den xyz-Achsen wider. Das Reflektieren von x kehrt den Strom um, das Reflektieren von y bringt ihn dorthin zurück, wo er begonnen hat, und das Reflektieren von z ändert den Solenoid nicht.

Da der Strom gleich ist, ist B gleich! Das B von einem Solenoid ändert sich also nicht unter Reflexion. Die Kraft auf ein Teilchen kann also nicht in Richtung von B wirken, da die Kraft bei Reflexion die Richtung umkehrt und B dies nicht tut. Die Kraft kann nur in Richtung einer Größe wirken, die die Richtung umkehrt, und die einfachste solche Größe ist v × B . Bei einer Reflexion kehrt v die Richtung um und B nicht, sodass sich die Lorentz-Kraft ordnungsgemäß umkehrt.

Dieses Argument geht von einer Reflexionssymmetrie aus, die eine Symmetrie des Elektromagnetismus ist, aber eigentlich keine grundlegende Symmetrie in unserem Universum ist. Das gleiche Reflexionsargument zeigt, dass die magnetische Ladung nicht richtig symmetrisch zur elektrischen Ladung ist, da die magnetische Ladung unter Reflexion das Vorzeichen ändert (spiegeln Sie alle Koordinaten mit der Ladung am Ursprung wider – das Feld bewegt sich an einen neuen Ort, zeigt aber auf denselben Richtung, so dass die Richtung der magnetischen Ladung umgekehrt wird). Diese Eigenschaft bedeutet, dass magnetische Monopole ein frühes Zeichen dafür waren, dass die Natur nicht paritätsinvariant ist, und könnte erklären, warum Dirac nicht überrascht war, als gezeigt wurde, dass die schwachen Wechselwirkungen die Parität verletzen.

Die andere Annahme ist, dass die Kraft die einfachste reflexionsinvariante Kombination von B und v ist. Wenn Sie die Idee aufgeben, dass die Kraft linear proportional zu B ist, gibt es kompliziertere Kombinationen, die auch funktionieren, um ein reflexionsinvariantes Kraftgesetz zu geben. Diese Kombinationen erfüllen im Allgemeinen nicht die Energieerhaltung.

Um eine automatische Energieerhaltung (und einen automatischen Phasenraum mit den symplektischen Eigenschaften) zu haben, sollten Sie Ihre Bewegungsgleichungen aus der Aktion ableiten.

Hamiltonsches Argument und Eichinvarianz

Das beste Argument stammt aus dem Konzept des Impulspotentials (oder Vektorpotentials). Als hätte die Energie eine potentielle Energie hinzugefügt, die ist e ϕ , das Momentum hat ein Potential hinzugefügt, das ist e EIN , wobei A das Vektorpotential ist.

Die Lagrangedichte für die Wechselwirkung ist

m EIN x ˙

Was den konjugierten Impuls ausmacht m v + e EIN , so dass die kinetische Energie ist ( p e EIN ) 2 2 m und die potentielle Energie ist e ϕ . Die Hamilton-Gleichungen für diese Energie ergeben das Lorentz-Kraftgesetz. Die Hamilton-Gleichungen lauten:

t p = x ( p e EIN ) 2 2 m + ϕ
t x = p e EIN m

Und das Kombinieren der Gleichungen zu einer Gleichung zweiter Ordnung für die Beschleunigung von x ergibt das Lorentz-Kraftgesetz. Dieselbe Ersetzung im Hamiltonian, p zu p e EIN , arbeitet in der Relativitätstheorie, um das korrekte vierdimensionale Lorentz-Kraftgesetz zu liefern.

Die Identifikation von B mit × EIN kann aus der Invarianz der Gleichungen unter Hinzufügen eines Gradienten zu A gerechtfertigt werden. Der klassisch physikalische Teil von A ist seine Kräuselung, und es ist sinnvoll, dies mit dem B in den Maxwell-Gleichungen zu identifizieren.

Dieses Argument ist grundsätzlich stichhaltig, weil es nicht von Reflexionsinvarianz abhängt (jedes Argument, das sich auf Reflexionssymmetrie stützt, ist wirklich falsch, da wir wissen, dass dies keine Symmetrie der Natur in irgendeinem fundamentalen Sinne ist) und quantenmechanisch korrekt ist, wenn Sie interpretieren die Eichinvarianz als Freiheit in der Neudefinition der lokalen Phase einer Wellenfunktion geladener Teilchen. Der einzige Nachteil ist, dass es eine gewisse Vertrautheit mit dem Hamilton-Prinzip erfordert.

Wie ist das intuitiv? Es gibt Mathe links, rechts und in der Mitte
@LarryHarson: Das ist intuitive Mathematik. Intuitiv bedeutet nicht unpräzise.
Es ist keine intuitive Mathematik, wenn sie komplizierter ist als der mathematische Ausdruck, den sie zu rechtfertigen versucht!!
@LarryHarson: Es ist komplizierter, aber weniger willkürlich.
"weniger willkürlich", das ist einfach unehrlicher Unsinn. Das Lorentz-Kraftgesetz ist wenig willkürlich, und Ihre Antwort verwirrt nur die Operation.
@LarryHarson: Was meinst du? Die Kraft ist senkrecht zu "v" und zu "B", und man muss dies ohne Relativitätstheorie argumentieren (da sie der Relativitätstheorie vorausgeht und sie inspiriert). Es wird nur dann nicht willkürlich, wenn Sie Relativität haben. Vielleicht ist es richtig, gleich zu Beginn „Relativität“ zu sagen, aber es wird nicht rechtfertigen, warum die Leute des 19. Jahrhunderts sicher waren, sie vor Einstein verstanden zu haben. Wenn Sie der Meinung sind, dass dies eine schlechte Antwort ist, stimme ich vielleicht zu - das Reflexionsproblem wird nicht mehr als grundlegend angesehen, und die Hamilton-Formulierung könnte auch willkürlich sein (aber ich denke nicht).
@ron Aus welchem ​​​​Papier, das vor der Relativitätstheorie liegt, stammt Ihre Ableitung?
@RonMaimon Wie inspiriert es die Relativitätstheorie?
Danke, dass du das gepostet hast, Ron. Niemand sollte erwarten, dass Technisches Schreiben einfach ist, und niemand sollte verlangen, alles beim ersten Lesen zu verstehen – man würde nichts lernen, wenn man nicht gefordert würde. Nicht alle Antworten funktionieren für alle - na und. In der Tat verlieren mich Ihre Antworten manchmal - na und - ich bin verdammt froh, dass Leute wie Sie sich die Mühe machen, die Sie in Ihre Posts stecken.

Die Lorentz-Kraft ist orthogonal zur Geschwindigkeit, was der Aussage entspricht, dass die Kraft keine Arbeit auf das geladene Teilchen ausübt; es ändert nur die Richtung der Geschwindigkeit, nicht ihre Größe.

Die Kraft ist auch orthogonal zum Magnetfeld. Sie folgt aus der Formel und diese Tatsache – und die ganze Formel – kann auf verschiedene Weise hergeleitet werden, z. B. aus der speziellen Relativitätstheorie.

Dieses Merkmal – die Kraft senkrecht zum Feld – unterscheidet das Magnetfeld von den elektrostatischen und Gravitationsfeldern. Insofern ist es anders: Anders als beim Elektrostatik- und Gravitationsfeld ist die Feldstärke kein Gradient irgendeines "skalaren Potentials". Aber es gibt kein Paradoxon. Unterschiedliche Wirkungen in der Natur können unterschiedlichen mathematischen Formeln folgen, und das tun sie oft.

Wenn Sie damit ein Problem haben, wissen Sie das einfach zu schätzen ( B x , B j , B z ) was wie ein Vektor aussieht, ist nur eine Abkürzung für ( F j z , F z x , F x j ) , drei Komponenten eines antisymmetrischen Tensors mit drei Indizes (die Komponenten sind Teilmengen des relativistischen Tensors F μ v die auch das elektrische Feld enthält).

Zum Beispiel, wenn B = ( 0 , 0 , B z ) , dann kann die dritte Komponente ungleich Null geschrieben werden als B z = F x j und anstelle eines Pfeils in der z -Richtung können Sie das Feld durch eine orientierte Schleife (mit einem Pfeil) in der visualisieren x j -Ebene (zu der die z -gerichteter Vektor ist normal). Es sind die gleichen Informationen.

Diese Schleife in der x j Flugzeug sagt Ihnen wirklich, was das Magnetfeld mit den geladenen Teilchen macht: Es dreht ihre Geschwindigkeit im Uhrzeigersinn (oder gegen den Uhrzeigersinn, je nach Vorzeichen). B z und die Ladung Q ) in dem x j -Flugzeug.

Es würde der Operation enorm helfen, wenn Sie zeigen könnten, wie die elektrische Kraft einer Ladung in einem statischen elektrischen Feld in einem anderen Rahmen, der nur wenige Zeilen dauern sollte, sowohl elektrisch als auch magnetisch wird.
Wie wäre es, wenn ich sagen würde ... bitte schön - mit Kirschen obendrauf?
Lieber Larry, ich bin mir nicht sicher, wie Ihre vorgeschlagene Ergänzung die ursprüngliche Frage beantworten würde. Für die Lorentz-Transformation des elektromagnetischen Felds siehe en.wikipedia.org/wiki/… - Die Kreuzprodukte in der Transformation hängen zwar mit der Frage zusammen, aber ich persönlich glaube, dass die Erklärung des Ursprungs und der Folgen dieser Transformationsgesetze mehr ist komplizierter als das, was das OP ursprünglich verlangte. Als Versuch, ein Problem zu entmystifizieren, ist es, als würde man ein Grippevirus mit dem HIV-Virus besiegen.
Ich schätze die Mühe, die Sie eindeutig in Ihre Antwort gesteckt haben. Leider geht das etwas über mein Wissen hinaus, aber ich werde mein Bestes versuchen, es zu verstehen. Vielen Dank.
Das OP bat um eine Erklärung einer bestimmten Tatsache. Die ersten drei Absätze wiederholen diese Tatsache nur, ohne zu erklären, warum sie wahr ist. Der vierte, fünfte und sechste Absatz sind wieder nur eine Feststellung der Tatsache, aber jetzt in schickerem mathematischem Gewand.

Hier ist ein Beispiel aus den Prinzipien der Elektrodynamik von Schwartz , basierend auf der Relativitätstheorie. (Habe ich diese Antwort gerade als "nicht intuitiv" disqualifiziert? Machen Sie trotzdem weiter ...)

Stellen Sie sich einen unendlichen, geraden Draht mit einem konstanten Strom vor, der aus einer gleichen Anzahl positiver und negativer Ladungen besteht, die in entgegengesetzte Richtungen fließen (die Nettoladungsdichte des Drahts ist also 0).

Fügen Sie nun ein Teilchen mit der Ladung q hinzu, das sich mit konstanter Geschwindigkeit v parallel zum Draht bewegt (Laborrahmen K). Welche Kraft wirkt auf das Teilchen?

Zur Beantwortung transformiert Schwartz das Problem auf das Ruhesystem K' des Teilchens. Wendet man den entsprechenden Lorentz-Boost auf den Ladungsstrom-Viervektor des Drahtes an, findet man, dass er in K' eine Ladungsdichte ungleich Null hat. Die Ladung wird daher durch ein elektrostatisches Feld zum Draht gezogen.

(Hier gibt es zwei Annahmen/empirische Tatsachen: 1) im Ruhesystem des Teilchens ist die Kraft durch das elektrische Feld gegeben, wie das Teilchen es sieht, und 2) weil die Ladungs- und Stromverteilung zeitunabhängig sind, kann man die elektrische berechnen Feld in K' durch den üblichen Integration-über-Ladungsdichte-Ansatz.)

Wenn man zum Laborrahmen zurückkehrt, findet man die Antwort auf die Frage, dass die sich bewegende Ladung eine Kraft senkrecht zu ihrer Geschwindigkeit spürt .

Wenn Sie nun das B-Feld für den Strom separat nach der üblichen Formel berechnen, finden Sie auch für die oben berechnete Kraft F =q vxB .

Natürlich ist das Obige nur ein Beispiel (und wäre nicht anwendbar, wenn sich das Teilchen auf den Draht zu bewegen würde, anstatt parallel dazu, da Annahme 2 verletzt würde.) Es gibt einen aufwändigeren Induktionsprozess, um zum vollständigen relativistischen Apparat zu gelangen . Das obige Beispiel belegt jedoch die Existenz einer Kraft senkrecht zur Geschwindigkeit eines geladenen Teilchens und zum Magnetfeld.

Dies ist das Beispiel, das ich in meinen Vorlesungen verwende. Obwohl ich mit einem geladenen Teilchen im Ruhezustand an einem stromführenden Draht beginne - keine Kraft. Schauen Sie sich dann die Felder im beweglichen Rahmen an - es gibt sowohl magnetische als auch elektrische Felder. Das Teilchen darf sich im beweglichen Rahmen nicht auf den Draht zubewegen, also muss es eine Kraft qvB geben und sie muss senkrecht zu v und B stehen. Wie Sie sagen – nur ein Beispiel, aber ein starkes.
@RobJeffries, schön! Danke für die Information. "Nulling"-Setups sind sehr attraktiv; ähm, du weißt was ich meine...

Ich gebe Ihnen hier ein sehr kurzes Argument auf der Grundlage der Quantenmechanik. Dieses Argument hat tatsächlich einen tiefgreifenden physikalischen und historischen Ursprung, den ich in der Fortsetzung darzulegen versuchen werde. In der Quantenmechanik lautet der Geschwindigkeitsoperator eines Teilchens in einem äußeren Magnetfeld (unter minimaler Kopplung):

v = p q EIN m

Wo EIN ist das Vektorpotential. Dies impliziert, dass das Magnetfeld unter Verwendung der kanonischen Kommutierungsbeziehungen gegeben ist:

B = ich m 2 v × v

Nun, es ist nicht schwer, das zu überprüfen

v . F ϵ ich j k [ v ich , [ v j , v k ] ]

die durch die Jacobi-Identität verschwindet ( F ist die Lorentzkraft), was die erforderliche Orthogonalitätsbeziehung ist.

Diese „Ableitung“ ist eigentlich Teil einer sehr interessanten Argumentation von Feynman, hinter der sich eine sehr interessante Geschichte verbirgt. Eigentlich erfordert das Argument keine Quantenmechanik, sondern nur den Begriff der Poisson-Klammer. Feynman nahm das Argument nicht ernst und veröffentlichte es nicht. Erst 1990 nach seinem Tod wurde dieses Argument (in seinem Namen) von Dyson veröffentlicht: F.Dyson,Am.J.Phys.58,209(1990).

Feynmans Argument ist sehr tiefgründig, weil es die Ableitung der gesamten Maxwell-Theorie (zusammen mit der Lorentz-Kraftgleichung) ausgehend von sehr einfachen und grundlegenden Annahmen ermöglicht:

  1. Die kanonischen Posisson-Klammern der Position und Geschwindigkeit.

  2. Minimale Kopplung: Die elektromagnetische Kraft auf ein geladenes Punktteilchen hängt nur von Position und Geschwindigkeit ab.

Bitte beachten Sie den einleitenden Abschnitt im folgenden Artikel von Carinena, Ibort, Marmo und Stern.

Eine der Verallgemeinerungen dieses Verfahrens ist die Ableitung der Yang-Mills-Gleichungen nach den gleichen Prinzipien

Dies ist im Vergleich zu der Ebene, auf der das OP die Frage gestellt hat, auf einer völlig falschen Ebene, und es hängt nur sehr tangential mit der Frage zusammen.
Okay, vielleicht ist dies nicht die fokussierteste Antwort (hey, ich würde NIE auf eine Tangente gehen, frag einfach meine Freunde :)), aber es ist auf jeden Fall fesselnd interessant und nicht ganz ohne Zusammenhang. Auf einer abstrakten Ebene IST es genau das, wonach das OP verlangt: tiefere, intuitive Motivation. Danke David, dass du das gepostet hast – ich habe etwas gelernt. Und es unterscheidet sich im Niveau nicht allzu sehr von den Ideen in Bens Buch (die ich auch verdammt interessant finde).

Ich nehme an, man könnte sich das in Bezug auf Quelle und Antwort vorstellen. Elektrische und magnetische Felder werden durch Quellen erzeugt, nämlich elektrische Ladungen und Ströme. Ströme kann man sich als relative Drift zwischen entgegengesetzt geladenen Teilchen vorstellen.

Betrachten wir ein Elektron mit Geschwindigkeit in der xy-Ebene (V ϕ ) unter dem Einfluss eines Magnetfeldes entlang + z ^ , oderB z . B herzustellen z , könnten wir einen kreisförmigen Stromdraht verwenden, I ϕ , in der xy-Ebene, die eine Vektorrichtung im positiven azimutalen Sinn (oder gegen den Uhrzeigersinn) hatte. Ströme werden durch die Bewegung positiver Ladungen definiert, also bewegen sich Elektronen in die entgegengesetzte Richtung.

Die Beschleunigung eines Elektrons durch den Einfluss von B z würde dazu führen, dass die Flugbahn des Partikels wie ein Kreis gegen den Uhrzeigersinn aussieht, ähnlich wie bei I ϕ zuvor erwähnt. Der Strom aufgrund dieses Elektrons, I e , ist in der entgegengesetzten Richtung zu I ϕ , oder ich ϕ ~ -ich e (Ignorieren Sie die Größen, kümmern Sie sich hier nur um das Vorzeichen).

Der Zweck dieser etwas ausführlichen Antwort besteht darin, zu veranschaulichen, dass die Reaktion eines Elektrons darin besteht, dem auf es einwirkenden Feld effektiv entgegenzuwirken. Es ähnelt dem Konzept der Induktion im Faradayschen Gesetz, bei dem ein Strom induziert wird, um zu versuchen, eine Änderung des Magnetflusses zu verhindern. Die Idee ist ähnlich mit dem Elektron, wo seine Umlaufbahn effektiv ein induzierter Strom ist und dieser Strom dem B entgegenwirkt z .

Dies sollte sinnvoll sein, da Energie/Impuls erhalten bleiben müssen. Man kann sich das auf zwei Arten vorstellen: B z Energie/Impuls verliert, um das Elektron zu beschleunigen oder B z verliert Energie/Impuls, weil der (durch die Umlaufbahn des Elektrons) induzierte Strom dagegen wirkt. Hmm, dieser letzte Teil ist verwirrender als ich zuerst dachte. Ich frage mich, ob das "oder" ein "und" sein sollte? Unabhängig davon reagiert das Teilchen auf das Feld und erzeugt einen effektiven Strom im entgegengesetzten Sinn zu dem, der das Feld erzeugt hat.

Sehr aufschlussreich. Vielen Dank! Konnte den letzten Kommentar nicht bearbeiten, musste ihn also entfernen und einen neuen eingeben.

Ich mag keine intuitiven Erklärungen, die nicht intuitiv sind! Intuitive Erklärungen können keine Formeln und Mathematik enthalten.

Es sollte eine Analogie geben, die, obwohl nicht ganz genau, dem Leser hilft, etwas hinter einer trockenen Formel oder einem Theorem zu spüren.

Ich habe lange nach einer intuitiven Erklärung für eine Lorentzkraft gesucht und jetzt habe ich eine gefunden, die mir sehr gefällt.

Beginnen wir mit einer Abbildung (unten), die die Lorentz-Kraft visualisiert als Wechselwirkung zwischen imaginären Magnetröhren zeigt.

Die Lorentz-Kraft visualisiert als Wechselwirkung zwischen magnetischen Rohren
(Quelle: conspiracyoflight.com )

Stellen Sie sich einen vor, der auf einen vertikalen Magneten schaut, Südteil auf der linken Seite und Nordteil auf der rechten Seite. Die Magnetlinien gehen von rechts nach links (N->S)

Stellen Sie sich nun eine positive Ladung vor, die sich vertikal durch die magnetischen Linien bewegt. Es erzeugt nach der Rechte-Hand-Regel ein Magnetfeld um sich herum. Die Linien dieses Feldes sind horizontal und gegen den Uhrzeigersinn.

Denken Sie daran, dass parallele magnetische Kraftlinien, die in die gleiche Richtung laufen, normalerweise die Folge einer Abstoßungskraft sind. Parallele magnetische Kraftlinien, die in entgegengesetzte Richtungen laufen, sind normalerweise die Folge einer Anziehungskraft.

Wenn Sie die Magnete und die sich bewegende Ladung in der Vertikalen betrachten, haben die magnetischen Linien (extern und erzeugte Ladung) auf der Rückseite (abgewandte Seite) die gleiche Richtung, die typischerweise durch eine Vorwärtskraft erzeugt wird. Auf der Vorderseite (Nahseite) sind die magnetischen Linien entgegengesetzt gerichtet, was normalerweise durch eine zusätzliche Vorwärtskraft erzeugt wird.

Infolgedessen erfährt das Teilchen eine Kraft von der fernen Seite zur nahen Seite, wie der dunkle Pfeil in der Abbildung zeigt.

Wenn schließlich die Kraft eine Komponente in der gleichen Richtung wie die Geschwindigkeit hätte, erzeugt die Kraft einen kontinuierlichen Geschwindigkeitsanstieg. Es wird für immer eine Erhöhung der kinetischen Energie erzeugt, da die Magnete nicht geladen werden müssen. Wenn die Kraft eine Komponente in der entgegengesetzten Richtung zur Geschwindigkeit hätte, werden die Ladungen anhalten und es gibt keinen möglichen elektrischen Strom innerhalb eines Magnetfelds.

Mit dem Aufkommen von Einsteins Behandlung des Elektromagnetismus wurden die magnetischen Kraftlinien zu einer imaginären Einheit verbannt. Es ist jedoch ein nützlicher Ansatz, um Konzepte zu erklären!

Quelle: https://www.conspiracyoflight.com/Lorentz/Lorentzforce.html

Ich glaube nicht, dass das wie geschrieben funktioniert. Feldlinien sind keine physischen Objekte, die sich anziehen und abstoßen, und die Ladung erfährt keine Kräfte von Feldern, die in einiger Entfernung von ihr auftreten.
Ich weiß, dass es keine magnetischen Linien gibt, weil die wahre Erklärung relativistisch ist. Es ist jedoch eine nützliche Abstraktion ( tpub.com/neets/book1/chapter1/1i.htm ), um die Intuition der Physik zu verbessern. Wenn Sie die obige Erklärung sorgfältig befolgen, werden Sie feststellen, dass sie mit der Handregelkonvention übereinstimmt.
Siehe faculty.polytechnic.org/physics/3%20A.P.%20PHYSICS%202009-2010/… . Verwenden Sie die Magnetfeldlinien-Analogie, um die richtige Kraftrichtung zu zeichnen. Sehen Sie sich die Beispiele für Magnete und Paralleldrähte an.
Ich weiß, dass es keine magnetischen Linien gibt, weil die wahre Erklärung relativistisch ist. Das ist nicht wirklich sinnvoll. Sie existieren, aber sie sind keine physischen Objekte. Die Powerpoint-Seite, auf die Sie verlinkt haben, unterstützt die Analyse in Ihrer Antwort tatsächlich nicht. Es beschreibt eine Regel, um festzustellen, ob sich physikalische Objekte anziehen oder abstoßen. Ihre Antwort spricht von anziehenden oder abstoßenden Feldlinien.
Ich spreche von der speziellen Relativitätstheorie, die den Magnetismus erklärt. Angenommen, zwei parallele Drähte A und B mit positivem Strom nach rechts. Holen Sie sich ein Elektron in A, der Abstand der Protonen in B verringert sich, sodass dieses Elektron angezogen wird. Holen Sie sich jetzt ein Proton in A, der Abstand der Elektronen in B verringert sich, sodass das Proton angezogen wird. Diese Kraft erklärt den Magnetismus.
Entschuldigung, ich spreche nicht fließend Englisch. Ich bin Brasilianer. Sie haben Recht. Magnetische Linien ziehen sich nicht an oder stoßen sich ab. Allerdings passiert das Gegenteil. Bei einer Abstoßungskraft sind die magnetischen Linien antiparallel. Eine Anziehungskraft zeigt normalerweise parallele Magnetlinien. Die magnetischen Feldlinien in der Abbildung geben einen Hinweis auf die Kraftrichtung. Tatsächlich ist es jedoch das Gegenteil. Die Kraft erzeugt dieses Magnetfeld gemäß der akzeptierten Konvention.
Ich bearbeite meine Antwort. Die Kombination von V, B und F in der bewegten Ladung hat immer konsistente Richtungen mit den Erklärungen. Ich weiß, dass es physikalisch nicht stimmt. Es ist nur etwas Intuition, die eine korrekte Richtung für die Lorentz-Kraft ergibt. Die auf der Lorentz-Kontraktion basierende spezielle Relativitätserklärung erzeugt auch eine richtige Richtung für F, B und V, aber für einen Laien ist das nicht so einfach.
Die Tatsache, dass die Relativitätstheorie den Magnetismus erklärt, hat nichts damit zu tun, ob Feldlinien real sind oder nicht. Es ist hier überhaupt nicht relevant.

Wie wäre es damit? Nehmen Sie an, dass die Lorentzkraft nicht senkrecht dazu steht v (das ist v hat eine von Null verschiedene Komponente parallel zu F ). Die Kraft wirkt auf die Ladung, wodurch sie beschleunigt wird ... was wiederum die Kraft erhöht ( q v B wo v ist die Komponente parallel zu F ) erhöht dies wiederum die Beschleunigung, die die Kraft erhöht und so weiter ins Unendliche. Dies würde offensichtlich den Energieerhaltungssatz verletzen und daher v muss senkrecht dazu stehen F . Das gleiche Argument erklärt warum B muss senkrecht dazu stehen v .

Nein, das schließt nicht aus, dass F eine Komponente antiparallel zu v haben könnte.
@RetardedPotential dann würde sich die Geschwindigkeit verlangsamen, ohne dass die Energie in Wärme umgewandelt wird. Verstößt immer noch gegen das erste Gesetz.
Auch wenn F eine antiparallele Komponente hätte, könnte ich einfach das Vorzeichen der Ladung ändern, oder nicht?
Wie erklärt dieses Argument, warum B muss senkrecht dazu stehen F ? (Du meintest F Am Ende? B muss nicht senkrecht stehen v .)
@Ryan Sie haben Recht, die Anklage umzudrehen, ich ziehe meinen anfänglichen Einwand zurück.
B muss senkrecht zu F sein, denn wenn dies nicht der Fall wäre, würde B eine Erhöhung der Geschwindigkeit verursachen, die dann eine Erhöhung der Kraft erzeugen würde und so weiter.
Ja, ich meinte, B muss am Ende senkrecht zu F sein. Vielen Dank
@ Ryan: Nein, es reicht aus, wenn F senkrecht zu v steht, dann nimmt die Geschwindigkeit nicht zu. Was hat es damit zu tun, dass F senkrecht auf B steht?
Ja, Sie haben Recht, ich denke, das obige Argument beweist nur, dass die Kraft senkrecht zur Geschwindigkeit sein muss.
Ich würde das in den Chat verschieben, aber ich kann nicht. Nachdem ich noch etwas darüber nachgedacht habe, kommt mir eine Idee. Wir haben festgestellt, dass F senkrecht auf v stehen muss. F kann also nur eine Richtungsänderung des Geschwindigkeitsvektors bewirken. Wenn das F nicht senkrecht zu B ist, wird die Kraft dazu führen, dass sich der Geschwindigkeitsvektor immer mehr mit dem B-Feld ausrichtet. Die Größe der Kraft ist jedoch proportional zu der Geschwindigkeitsvektorkomponente senkrecht zu v. Wenn also F nicht senkrecht zu B ist, dann bewirkt die Kraft, dass v mit B ausgerichtet wird, wodurch die Größe der Kraft verringert wird.

Lassen Sie sich nicht von der „seltsamen“ Definition des Magnetfelds täuschen B . Stellen Sie sich zwei Teilchen vor, die sich mit Geschwindigkeiten bewegen v 1 und v 2 , getrennt durch einen Abstand d (relativ nahe, um Verzögerungseffekte in der Feldausbreitung zu ignorieren) und derart, dass im Moment der Trennungsvektor zwischen ihnen liegt r = d   u ^ 12 (Sein u ^ 12 ein Einheitsvektor). Die Kraft von Teilchen 1 auf Teilchen 2 ist:

F 1 2 = q 2 v 2 × B 1 = μ q 2 q 1 4 π   v 2 × ( v 1 × u ^ 12 ) d 2

während die Kraft von Teilchen 2 auf Teilchen 1 ist:

F 2 1 = q 1 v 1 × B 2 = μ q 2 q 1 4 π   v 1 × ( v 2 × ( u ^ 12 ) ) d 2

Durch die Verwendung der Vektoridentität a × ( b × c ) = ( a × c ) × b = ( a × b ) × c , ist ersichtlich, dass die erste Kraft F 1 2 liegt in der Ebene, die durch gebildet wird u ^ 12 und v 1 . Andererseits die zweite Kraft F 2 1 liegt in der Ebene, die durch gebildet wird u ^ 12 und v 2 .

Daher liegt die Kraft jedes Teilchens auf der Ebene, die durch die Trennung und seine eigene Geschwindigkeit definiert ist: Es ist ein bisschen so, als ob jedes Teilchen in die Richtung seiner eigenen Geschwindigkeit und in die Richtung zum anderen Teilchen drückt !

Für die Intuition können Sie vielleicht aus einer experimentellen Perspektive darüber nachdenken. Wenn Sie ein geladenes Teilchen betrachten, das sich in einem gleichmäßigen Magnetfeld bewegt, ist die Bewegung kreisförmig. Betrachten Sie zum Beispiel den Weg eines Teilchens in einer Nebelkammer.

Energetische Figur in der Nebelkammer

Sie können dann beispielsweise den Radius messen, und da er sich kreisförmig bewegt, stellen Sie fest, dass eine nach innen gerichtete Kraft vorhanden sein muss, die im rechten Winkel zur Richtung der Geschwindigkeitsbewegung steht.

Geben Sie hier die Bildbeschreibung ein

Da im Mittelpunkt des Kreises keine andere Ladung vorhanden ist, um die das Teilchen kreisen kann, oder Masse oder Gravitationskraft, damit das Teilchen um etwas kreist, muss die Kraft erzeugt werden (wir nennen es die Lorentz-Kraft).

Wenn Sie an Ihre klassische Mechanik zurückdenken, kennen Sie das

Sie können andere Experimente durchführen, z. B. die Masse des geladenen Teilchens ändern und sehen, dass sich der Radius des Kreises ändert. Wenn Sie das tun, finden Sie den Radius ist

ρ = m v q B

oder das Nachdenken über die Beschleunigung für ein Masseteilchen im Kreis ist m v 2 ρ Sie kommen mit der Kraft

f = q v B = m v 2 ρ

An diesem Punkt ist f= qvB dasselbe wie qvBsin(90), da der Winkel 90 Grad beträgt und die Definition des Kreuzprodukts geschrieben werden kann als

v × B = | v | | B | Sünde θ n ^

Und Sie machen noch ein paar Experimente und finden heraus, dass der Winkel zwischen der Geschwindigkeit des Teilchens und dem Magnetfeld wichtig ist, und das Schreiben der Vektorgleichungen als Kreuzprodukt ist eine nette Abkürzung dafür.

So F = q ( v × B )

Geben Sie hier die Bildbeschreibung ein

Das aufgeladene Teil würde niemals an Ort und Stelle bleiben. Sie versuchen, ohne die gesamte Gleichung zu lösen, aber Ihre Antwort sollte für ein funktionierendes Modell so aussehen. Stattdessen würdest du die positive Monopolscheibe durch dein positiv geladenes Teilchen ersetzen.